LSAT and Law School Admissions Forum

Get expert LSAT preparation and law school admissions advice from PowerScore Test Preparation.

User avatar
 Dave Killoran
PowerScore Staff
  • PowerScore Staff
  • Posts: 5853
  • Joined: Mar 25, 2011
|
#44159
Complete Question Explanation
(The complete setup for this game can be found here: lsat/viewtopic.php?t=13729)

The correct answer choice is (B)

From the basic physical constraints of the game, Philadelphia can only connect with four other cities: Honolulu, Montreal, Toronto, and Vancouver. From the fourth rule, we know that Philadelphia cannot connect to both Toronto and Vancouver, and thus, the options for Philadelphia are H, M, and V/T. The question stem in #12 indicates that exactly three cities are connected with Philadelphia. Hence, we know we are selecting from the group of H, M, and V/T. But, because of the action of the third rule, when Philadelphia is connected with Honolulu, then Philadelphia must also be connected with Toronto. We can then determine in question #12 that Philadelphia is connected only to H, M, and T, as follows:
J03_Game_#3_#12_diagram 1.png
Because P cannot connect with V, we can eliminate answer choice (E). Also, because M is now connected to P, from the second rule we know that M cannot connect with any other city. Thus, answer choices (A), (C), and (D) can be eliminated. Therefore, answer choice (B) is proven correct by process of elimination.
You do not have the required permissions to view the files attached to this post.

Get the most out of your LSAT Prep Plus subscription.

Analyze and track your performance with our Testing and Analytics Package.